Theorem Definition and 1000 Threads

In mathematics and logic, a theorem is a non-self-evident statement that has been proven to be true, either on the basis of generally accepted statements such as axioms or on the basis of previously established statements such as other theorems. A theorem is hence a logical consequence of the axioms, with a proof of the theorem being a logical argument which establishes its truth through the inference rules of a deductive system. As a result, the proof of a theorem is often interpreted as justification of the truth of the theorem statement. In light of the requirement that theorems be proved, the concept of a theorem is fundamentally deductive, in contrast to the notion of a scientific law, which is experimental.Many mathematical theorems are conditional statements, whose proofs deduce conclusions from conditions known as hypotheses or premises. In light of the interpretation of proof as justification of truth, the conclusion is often viewed as a necessary consequence of the hypotheses. Namely, that the conclusion is true in case the hypotheses are true—without any further assumptions. However, the conditional could also be interpreted differently in certain deductive systems, depending on the meanings assigned to the derivation rules and the conditional symbol (e.g., non-classical logic).
Although theorems can be written in a completely symbolic form (e.g., as propositions in propositional calculus), they are often expressed informally in a natural language such as English for better readability. The same is true of proofs, which are often expressed as logically organized and clearly worded informal arguments, intended to convince readers of the truth of the statement of the theorem beyond any doubt, and from which a formal symbolic proof can in principle be constructed.
In addition to the better readability, informal arguments are typically easier to check than purely symbolic ones—indeed, many mathematicians would express a preference for a proof that not only demonstrates the validity of a theorem, but also explains in some way why it is obviously true. In some cases, one might even be able to substantiate a theorem by using a picture as its proof.
Because theorems lie at the core of mathematics, they are also central to its aesthetics. Theorems are often described as being "trivial", or "difficult", or "deep", or even "beautiful". These subjective judgments vary not only from person to person, but also with time and culture: for example, as a proof is obtained, simplified or better understood, a theorem that was once difficult may become trivial. On the other hand, a deep theorem may be stated simply, but its proof may involve surprising and subtle connections between disparate areas of mathematics. Fermat's Last Theorem is a particularly well-known example of such a theorem.

View More On Wikipedia.org
  1. e2m2a

    A Clarification of Mihăilescu's Theorem (Catalan's Conjecture)

    Mihăilescu's theorem proves that Catalan's conjecture is true. That is for x^a - y^b = 1, the only possible solution in naturual numbers for this equation is x=3, a=2, y=2, b=3. What is not clear to me is this. Does Mihăilescu's theorem prove that the difference between any other two...
  2. C

    Simple Induction Induction proof of Polynomial Division Theorem

    Theorem: Let ## f(x), g(x) \in \mathbb{F}[ x] ## by polynomials, s.t. the degree of ## g(x) ## is at least ## 1 ##. Then: there are polynomials ## q(x), r(x) \in \mathbb{F}[ x] ## s.t. 1. ## f(x)=q(x) \cdot g(x)+r(x) ## or 2. the degree of ## r(x) ## is less than the degree of ## g(x) ## Proof...
  3. M

    Bayes Theorem: coin flips and posterior predictive distribution

    Hi, I was attempting the following question and just wanted to check whether my working was correct: Question: A bag has three coins in it which are visually indistinguishable, but when flipped, one coin has a 10% chance of coming up heads, another as a 30% chance of coming up heads, and the...
  4. e2m2a

    A Prime Factorization Theorem and Number Systems

    If you go to "The Abel Prize Interview 2016 with Andrew Wiles" on YouTube, there is a statement made by Andrew Wiles beginning at about 4:10 and ending about 4:54 where he mentions there are some new number systems possible where the fundamental theorem of arithmetic does not hold. I don't...
  5. A.T.

    B Intermediate Axis Theorem - Intuitive Explanation

    A simulation/animation/explanation based on the inertial frame only: The previous videos referenced there are here: See also this post for context on the Veritasium video: https://mathoverflow.net/a/82020 Note to mods: The previous thread is not open anymore so I opened a new one. Feel free...
  6. L

    Stokes' theorem gives different results

    Given surface ##S## in ##\mathbb{R}^3##: $$ z = 5-x^2-y^2, 1<z<4 $$ For a vector field ##\mathbf{A} = (3y, -xz, yz^2)##. I'm trying to calculate the surface flux of the curl of the vector field ##\int \nabla \times \mathbf{A} \cdot d\mathbf{S}##. By Stokes's theorem, this should be equal the...
  7. S

    I How to visualize division in the Odds form of Bayes's Theorem?

    Here I'm asking solely about the circle pictograms. Please eschew referring to, or using, numbers as much as possible. Please explain using solely the circle pictograms. Undeniably, I'm NOT asking about how to divide numbers. I don't understand 1. How do I "visually" divide Circle 1...
  8. S

    MHB How to visualize division in the Odds form of Bayes's Theorem?

    I saw this question at https://math.codidact.com/posts/283253.
  9. kshitij

    How to find centroid of a hemisphere using Pappus's centroid theorem?

    I recently learned how to calculate the centroid of a semi-circular ring of radius ##r## using Pappus's centroid theorem as ##\begin{align} &4 \pi r^2=(2 \pi d)(\pi r)\nonumber\\ &d=\frac {2r}{\pi}\nonumber \end{align}## Where ##d## is the distance of center of mass of the ring from its base...
  10. P

    I Confusion over applying the 1st uniqueness theorem to charged regions

    1. For regions that contain charge density, does the 1st uniqueness theorem still apply? 2. For regions that contain charge density, does the 'no local extrema' implication of Laplace's equation still apply? I think not, since the relevant equation now is Poisson's equation. Furthermore...
  11. Leo Liu

    How to use the divergence theorem to solve this question

    The correct answer is ##\frac{\pi a^2 h} 2## by using the standard approach. However when I tried using the divergence theorem to solve this problem, I got a different answer. My work is as follows: $$\iint_S \vec F\cdot\hat n\, dS = \iiint_D \nabla\cdot\vec F\,dV$$ $$= \iiint_D \frac{\partial...
  12. LCSphysicist

    Singularities when applying Stokes' theorem

    It is more or less a generic problem of stokes theorem: ##\int_{\gamma} F dr##, where ##F = (-y/(x²+y²) + z,x/(x²+y²),ln(2+z^10))## and gamma is the intersection of ##z=y^2, x^2 + y^2 = 9## oriented in such way that its projection in xy is traveled clockwise. So, i decided to apply stokes...
  13. T

    A Noether's theorem for finite Hamiltonian systems

    The Noether's theorem for finite Hamiltonian systems says that: My question is: If I know a symmetry how can I write the first integral?
  14. Twigg

    A Proof of Classical Fluctuation-Dissipation Theorem

    Sorry if there's latex errors. My internet connection is so bad I can't preview. Here's the wikipedia proof I'm referring to. I'm fine with the steps up to $$W(x,0) = W_0 (x) [1 + \beta f_0 (x(0) - \langle x \rangle_0) ]$$ where ##W(x,t)## is the probability density of finding the system at...
  15. A

    Why can't I use the divergence theorem?

    Greetings! here is the following exercice I understand that when we follow the traditional approach, (prametrization of the surface) we got the answer which is 8/3 But why the divergence theorem can not be used in our case? (I know it's a trap here) thank you!
  16. Strilanc

    I A tool for checking claims of violating Bell's theorem

    I just wanted to point out a resource useful for dealing with claims of violating Bell's theorems. You can point the claimant at https://algassert.com/quantum/2015/10/11/Bell-Tests-vs-No-Communication.html and say "I won't believe you unless you can make the 'Write Your Own Classical CHSH...
  17. HrvojeDjurdjevic

    I Is the proof of the KRK endgame theorem rigorous and original?

    The articles: https://lmcs.episciences.org/5328/pdf http://argo.matf.bg.ac.rs/publications/2013/2013-icga-krk-sat.pdf http://archive.ceciis.foi.hr/app/public/conferences/1/papers2012/dkb3.pdf KRK endgame is a win for white regardless of starting position, with the trivial drawing exception in...
  18. facenian

    I Munkres Chapter 5: Problem involving the Tychonoff Theorem

    Hi, In Chapter 5 Munkres proves the Tychonoff Theorem and after proving the theorem the first exercise is: Let ##X## be a space. Let ##\mathcal{D}## be a collection of subsets of ##X## that is maximal with respect to finite intersection property (a) Show that ##x\in\overline{D}## for every...
  19. F

    MHB Proving Relatively Compactness in C([a, b]) using Arzelá-Ascoli Theorem

    I don't know how to solve this proof Prove that a set $M \subset C([a, b])$ for which there exist $m. L> 0$ and $x_0 \in [a; b]$ such that $|f(x_0)| \leq{} m$ for all $f \in M$ and $|f(x)-f(y)| \leq{} L |x-y|$ for all $f\in M$ and for all $x,y \in [a,b]$ is relatively compact in $C([a, b])$ My...
  20. M

    I Model used to refute Bell's theorem

    I’m looking over a recent paper mentioned in another thread. It claims to refute Bell’s theorem. At first glance, the model presented in the paper doesn’t appear consistent with QM. Here’s a simple example. Suppose we set both polarizers to the same angle ##\alpha = \pi /4##. In the model...
  21. O

    I Need some help with trace calculation in an index theorem

    I hate to create a thread for a step in a calculation, by I don't know what else to do. I'm having a lot of trouble reproducing E. Weinberg's index calculation (found here https://inspirehep.net/literature/7539) that gives the dimension of the moduli space generated by BPS solutions in the...
  22. E

    I Bell's theorem claimed to be refuted - paper published by EPL

    The Paper “On a contextual model refuting Bell’s theorem” has now been published by the journal EPL (Europhysics Letters) and is available under https://iopscience.iop.org/article/10.1209/0295-5075/134/10004 In this paper a contextual realistic model is presented which correctly predicts...
  23. P

    What exactly does the Work-Energy theorem say?

    My research leads to 2 slightly different equations. See equations 1 and 2 attached. Also, for equation 1 should Wext be Wnet ext ?
  24. I

    I Question about No-Cloning Theorem and State Tomography

    Hi, I have a question, or am looking for clarification, about the no-cloning theorem and state tomography. My understanding is that the theorem states one cannot make an exact copy of a quantum state. I was also reading about state state tomography where it was said* 'On the other hand, the...
  25. JackyCheukKi

    Work and Energy on a Slope - How Does a Block Move Up with Zero Net Work?

    Guys, I have a problem that really needs you guys to help, I know it is a stupid question but please bear with me: Context: You have a block on a slope(has friction) you use a string to pull the block up with constant speed. Problem: So according to the network theorem, the work net is equal...
  26. Eclair_de_XII

    B Is this a valid proof for the Extreme Value Theorem?

    If ##f## is a constant function, then choose any point ##x_0##. For any ##x\in K##, ##f(x_0)\geq f(x)## and there is a point ##x_0\in K## s.t. ##f(x_0)=\sup f(K)=\sup\{f(x_0)\}=f(x_0)##. Now assume that ##f## is not a constant function. Construct a sequence of points ##x_n\in K## as follows...
  27. Andrei0408

    Calculate the voltage across a resistor using Thevenin's Theorem

    I'm thinking I should apply Thevenin's Theorem to find the voltage, but I need to find I_D in order to be able to calculate U_S.
  28. A

    A Levitron and Earnshaw’s theorem.

    The Earnshaw’s theorem comes directly from Maxwell equation so it should be unavoidable in any classical situation. The theorem usually disallows magnetic levitation. However, there are loopholes. Quoting wikipedia "Earnshaw's theorem has no exceptions for non-moving permanent ferromagnets...
  29. C

    I Proof check for Plancherel's theorem (Fourier Transform version)

    I'm trying to prove Plancherel's theorem for functions $$f\in L^1\cap L^2(\mathbb{R})$$. I've included below my attempt and I would really appreciate it if someone could check this for me please, and give me any feedback they might have. **Note:** I am working with a slightly different...
  30. Demystifier

    A Assumptions of the Bell theorem

    Loosely speaking, the Bell theorem says that any theory making the same measurable predictions as QM must necessarily be "nonlocal" in the Bell sense. (Here Bell locality is different from other notions of locality such as signal locality or locality of the Lagrangian. By the Bell theorem, I...
  31. R

    Some Questions About Thevenin's Theorem

    (1) We want to find the voltage across ##R_L## (2) We remove the load and label the terminals ##V_T## (3) The equivalent network of (2) So basically the voltage across the load is ##V_{th}## but when we find the equivalent network we put that same voltage behind a Thevenin...
  32. R

    Circuit Analysis Theorem For the Number of Independent Equations

    I attached a screenshot of the book (sorry no pdf available for this book). Right above the somewhat central line they give the theorem that if there are m currents and n nodes, then there will be n - 1 independent equations from the current law and m - n - 1 from the voltage law. I count 4...
  33. W

    B Extending the Fundamental Theorem of Arithmetic to the rationals

    The Fundamental Theorem of Arithmetic essentially states that any positive whole number n can be written as: ##n = p_1^{a_1} \cdot p_2^{a_2} \cdot p_3^{a_3} \cdot \dots## where ##p_1##, ##p_2##, ##p_3##, etc. are all the primes, and ##a_1##, ##a_2##, ##a_3##, etc. are non-negative integers...
  34. N

    MHB Proving the Squeeze Theorem - Finding Limits Using Squeeze Theorem

    Use the Squeeze Theorem to find the limit. lim [x^2 • (1 - cos(1/x)] x--> 0 Let me see. -1 ≤ cos (1/x) ≤ 1 -x^2 ≤ x^2 • [1 - cos(1/x)] ≤ x^2 -|x^2| ≤ x^2 • [1 - cos(1/x)] ≤ |x^2| lim -|x^2| as x tends to 0 = 0. lim |x^2| as x tends to 0 = 0. . By the Squeeze Theorem, [x^2 • (1 -...
  35. N

    MHB How to Use the Squeeze Theorem to Find This Limit?

    Use the Squeeze Theorem to find the limit. lim (x^2 • sin(1/x)) x--> 0 Let me see. -1 ≤ sin (1/x) ≤ 1 -x^2 ≤ x^2 • sin(1/x) ≤ x^2 -|x^2| ≤ x^2 • sin(1/x) ≤ |x^2| lim -|x^2| as x tends to 0 = 0. lim |x^2| as x tends to 0 = 0. . By the Squeeze Theorem, x^2 • sin(1/x) was squeezed between...
  36. N

    MHB Is the Squeeze Theorem Correctly Applied Here?

    If 0 ≤ f(x) ≤ 1 for every x, show that lim [x^2 • f(x)] = 0. x--> 0 Let me see. 0 ≤ f(x) ≤ 1 Multiply all terms by x^2. 0 • x^2 ≤ x^2• f(x) ≤ 1 • x^2 0 ≤ x^2 • f(x) ≤ x^2 Is this right so far? If correct, what's next?
  37. Eclair_de_XII

    Attempting to prove the Intermediate Value Theorem

    Proof goes like this: (1) Prove the existence of open intervals centered around the end-points of the domain such that the image of the points in these intervals through ##f## has the same sign as the image of the end-point through ##f##. In other words, prove that there is a ##\delta>0## such...
  38. N

    MHB Why Doesn't the Intermediate Value Theorem Apply to ln(x^2 + 2) on [−2, 2]?

    Explain why the Intermediate Value Theorem gives no information about the zeros of the function f(x) = ln(x^2 + 2) on the interval [−2, 2]. Let me see. Let x = -2. f(-2) = ln((-2)^2 + 2) f(-2) = ln(4 + 2) f(-2) = ln (6). This is a positive value. When I let x be 2, I get the same answer...
  39. S

    Engineering Norton's Theorem -- Help checking my work please

    So I've just learned Norton's Theorem and I have this problem on my homework assignment that is wrong. I've checked the answer with a circuit simulator(PSPICE) and the simulation said that V0 should be a drop of 2V. However, my simplified circuit shows a voltage drop of 4V. I have been staring...
  40. E

    B Confusion about Divergence Theorem Step in Tong's Notes

    I wanted to ask about a step I couldn't understand in Tong's notes$$\int_M d^n x \partial_{\mu}(\sqrt{g} X^{\mu}) = \int_{\partial M} d^{n-1}x \sqrt{\gamma N^2} X^n = \int_{\partial M} d^{n-1}x \sqrt{\gamma} n_{\mu} X^{\mu}$$we're told that in these coordinates ##\partial M## is a surface of...
  41. F

    I Can we discover new theorems by analyzing their proofs?

    In Elementary Geometry we can use drawing figure to guess the geometry theorem.How can we guess a theorem in Math in general?
  42. B

    Bayes' theorem and disease prevalence

    Hello at all! I have to solve this exercise: A tampon diagnostic test provides 1% positive results. The positive predictive values (probabilities of positive test disease) and negative (absence disease given negative test) are respectively 0.95 and 0.98. What is the prevalence of the disease...
  43. LCSphysicist

    Check the spectral theorem for this matrix

    I found three projection operators $$P_{1}= \begin{pmatrix} 1/2 & & \\ & -\sqrt{2}/2 & \\ & & 1/2 \end{pmatrix}$$ $$P_{2}= \begin{pmatrix} 1/2 & & \\ & \sqrt{2}/2 & \\ & & 1/2 \end{pmatrix}$$ $$P_{3}= \begin{pmatrix} -1/\sqrt{2} & & \\ & & \\ & & 1/\sqrt{2} \end{pmatrix}$$ From this five...
  44. S

    Engineering Please help me with this circuit question using the superposition theorem

    I have tried many times to solve this network, but can't understand how to get current in each resistors by superposition theorem. Please help me to solve and find currents in each 3 resistors with solution. Note:- The figure is attached below.
  45. A

    B 4-colours theorem can be proved visually

    The 4-colour theorem states that the maximum number of colours required to paint a map is 4. The proof requires exhaustive computation with a help of a computer. But I thought that one can visually prove the theorem in the following way; If one replaces the map with a graph where each region...
  46. J

    I Does Poynting's Theorem only involve external fields?

    Poynting's Theorem (https://en.wikipedia.org/wiki/Poynting's_theorem) says: The rate of energy transfer (per unit volume) from a region of space equals the rate of work done on a charge distribution plus the energy flux leaving that region. $$-\frac{\partial u}{\partial...
  47. DaTario

    I ##(p^k -1) \equiv X \mbox{(mod p)}## via Wilson's theorem

    Hi All, being ##p## a prime number, is there a way to solve the congruence ##(p^k-1)! \equiv X \mbox{ (mod p)}## for ##X## using Wilson's theorem: $$ (p-1)! \equiv -1 \mbox{(mod p)} $$?
  48. Demystifier

    A Is the Ensemble Interpretation Inconsistent with the PBR Theorem?

    [Moderator's note: Spun off from another thread due to topic and subforum change.] I think Ballentine's interpretation is ruled out by the PBR theorem. Maybe we could discuss that?
  49. Decimal

    I Does the Wigner-Eckart theorem require good quantum numbers?

    I have a question related to the following passage in the quantum mechanical scattering textbook by Taylor, Here Taylor makes the choice to use a basis of total angular momentum eigenvectors instead of using the simple tensor product given in the first equation above (6.47). I understand that...
Back
Top